ChaseDream

标题: 两道 LSAT求教 [打印本页]

作者: qiqi_cola    时间: 2003-9-10 22:18
标题: 两道 LSAT求教
If the city council maintains spending at the same level as this year's it can be expected to levy a sales tax of 2 percent next year.
Thus,if the council levies a higher tax,it will be because the council is increasing its expenditure.

If the companies in the state do not increase their worker's wage this year,the prices they charge for their goods can be expected to be much the same as they were last year.Thus,if the companies do increase prices,it will be because they have increased wages.

我感觉这两个是SIMILAR,但我说不出来哪里是SIMILAR,XDJM帮忙解答一下。





The public in the United States has in the past been conditioned to support a substantial defense budget by the threat of confrontation with the Eastern bloc.Now that that threat is dissolving,along with the Eastern bloc itself,it is doubtful whether the public can be persuaded to support an adequate defense budget.

Which one of the following indicates a weakness in the position expressed above?

A It presupposes that public opinion can be manipulated indefinitely,without the public's becoming aware of that manipulation.

B  It refers to past and present events that do not have a causal connection with public support of the budget.

C  It assumes as fact what it seeks to establish by reasoning.

D  It fails to give any reason for the judgement it reaches.

E  It hinges on the term adequate the precise meaning of which requires reevaluation in the new context.


另外再问问,大家做ASSUMPTION类型的方法,我老错。
作者: bigmouse    时间: 2003-9-10 22:32
我只对第一个问题说一下我的理解!
A:If the city council maintains spending at the same level as this year's
B:it can be expected to levy a sales tax of 2 percent next year.(A->B)
thus
非B:if the council levies a higher tax
非A:it will be because the council is increasing its expenditure.(非B->非A)
答案和这个推理一致。不知我的解释是否准确,请NN指点一下!
作者: polarbear223    时间: 2003-9-11 03:32
第三题,

是不是E? 题中先说是:substantial defense budget 后说:
adequate defense budget
所以我选E

请指教
作者: Mission    时间: 2003-9-11 05:24
question 3:

    feel it should be E, but not quite clear about how to interpret  the sentence.

    I think the logic in this question should be

   Threaten by the Eastern bloc -------- people are convinced to support defense budget
    ( A ----- B )
   No threat from the Eastern Bloc --------- people will not be convinced to support defense budget   ( not A -------- not B )

    So there is a logic fault in it.

    Am i right? Any NN give some suggestion?




欢迎光临 ChaseDream (https://forum.chasedream.com/) Powered by Discuz! X3.3